Last visit was: 26 Apr 2024, 22:13 It is currently 26 Apr 2024, 22:13

Close
GMAT Club Daily Prep
Thank you for using the timer - this advanced tool can estimate your performance and suggest more practice questions. We have subscribed you to Daily Prep Questions via email.

Customized
for You

we will pick new questions that match your level based on your Timer History

Track
Your Progress

every week, we’ll send you an estimated GMAT score based on your performance

Practice
Pays

we will pick new questions that match your level based on your Timer History
Not interested in getting valuable practice questions and articles delivered to your email? No problem, unsubscribe here.
Close
Request Expert Reply
Confirm Cancel
SORT BY:
Date
Tags:
Show Tags
Hide Tags
User avatar
Intern
Intern
Joined: 25 May 2013
Posts: 24
Own Kudos [?]: 105 [94]
Given Kudos: 16
Send PM
Most Helpful Reply
Math Expert
Joined: 02 Sep 2009
Posts: 92948
Own Kudos [?]: 619276 [31]
Given Kudos: 81609
Send PM
GMAT Club Legend
GMAT Club Legend
Joined: 19 Dec 2014
Status:GMAT Assassin/Co-Founder
Affiliations: EMPOWERgmat
Posts: 21846
Own Kudos [?]: 11669 [8]
Given Kudos: 450
Location: United States (CA)
GMAT 1: 800 Q51 V49
GRE 1: Q170 V170
Send PM
General Discussion
Math Expert
Joined: 02 Sep 2009
Posts: 92948
Own Kudos [?]: 619276 [8]
Given Kudos: 81609
Send PM
The next number in a certain sequence is defined by multiply [#permalink]
2
Kudos
6
Bookmarks
Expert Reply
Bunuel wrote:
The next number in a certain sequence is defined by multiplying the previous term by some positive constant k, where k ≠ 1. How many of the first nine terms in this sequence are greater than 1?

Given that:
\(a_2=a_1*k\)
\(a_3=a_1*k^2\)
\(a_4=a_1*k^3\)
...
\(a_n=a_1*k^{n-1}\)

Also given that k>0 and n=9.

(1) The ninth term in this sequence is 81 --> \(a_9=a_1*k^8=81\). If \(a_1=1\), then all but \(a_1\) will be greater than 1, but if \(a_1=2\), then all will be greater than 1. Not sufficient.

(2) The fifth term in this sequence is 1 --> \(a_5=a_1*k^4=1\). Now, if \(a_1<1\), then \(k>1\), and all terms from \(a_5\) (\(a_6\), \(a_7\), \(a_8\), and \(a_9\)), so 4 terms will be greater than 1 AND if \(a_1>1\), then \(k<1\), and all terms till \(a_5\) (\(a_2\), \(a_3\), \(a_4\), and \(a_5\)), so again 4 terms will be greater than 1. Sufficient.

Answer: B.


Similar questions to practice:
https://gmatclub.com/forum/m15-71960.html
https://gmatclub.com/forum/in-the-sequen ... 26119.html
https://gmatclub.com/forum/if-m-is-a-seq ... 32988.html
https://gmatclub.com/forum/if-a1-a2-a3-a ... 29753.html
https://gmatclub.com/forum/the-numbers-a ... 06213.html
https://gmatclub.com/forum/the-sequence- ... 03947.html
Manager
Manager
Joined: 22 Feb 2016
Posts: 67
Own Kudos [?]: 52 [0]
Given Kudos: 208
Location: India
Concentration: Economics, Healthcare
GMAT 1: 690 Q42 V47
GMAT 2: 710 Q47 V39
GPA: 3.57
Send PM
The next number in a certain sequence is defined by multiply [#permalink]
Bunuel wrote:
The next number in a certain sequence is defined by multiplying the previous term by some positive constant k, where k ≠ 1. How many of the first nine terms in this sequence are greater than 1?

Given that:
\(a_2=a_1*k\)
\(a_3=a_1*k^2\)
\(a_4=a_1*k^3\)
...
\(a_n=a_1*k^{n-1}\)

Also given that k>0 and n=9.

(1) The ninth term in this sequence is 81 --> \(a_9=a_1*k^8=81\). If \(a_1=1\), then all but \(a_1\) will be greater than 1, but if \(a_1=2\), then all will be greater than 1. Not sufficient.

(2) The fifth term in this sequence is 1 --> \(a_5=a_1*k^4=1\). Now, if \(a_1<1\), then \(k>1\), and all terms from \(a_5\) (\(a_6\), \(a_7\), \(a_8\), and \(a_9\)), so 4 terms will be greater than 1 AND if \(a_1>1\), then \(k<1\), and all terms till \(a_5\) (\(a_2\), \(a_3\), \(a_4\), and \(a_5\)), so again 4 terms will be greater than 1. Sufficient.

Answer: B.


Just wondering,

can K take any positive fractional value or does it need to be an interger.

It does not explicitly mentions that K has to be an integer. I agree the answer will be the same in both the cases however just want to clarify in case we face a similar situation during the main exam.
GMAT Club Legend
GMAT Club Legend
Joined: 19 Dec 2014
Status:GMAT Assassin/Co-Founder
Affiliations: EMPOWERgmat
Posts: 21846
Own Kudos [?]: 11669 [0]
Given Kudos: 450
Location: United States (CA)
GMAT 1: 800 Q51 V49
GRE 1: Q170 V170
Send PM
Re: The next number in a certain sequence is defined by multiply [#permalink]
Expert Reply
Hi AmritaSarkar89,

In my solution (above), I discuss this exact issue. GMAT questions are very carefully worded, so you have to pay attention to the details (what those details state and DON'T state). Here, we're told that K is a POSITIVE CONSTANT. That's does not mean that K is necessarily an integer (in the same way that stating N > 0 does not mean that N is necessarily an integer). Be mindful of these details when working through GMAT questions and make sure to write everything on the pad when you're working (so that you don't forget anything).

GMAT assassins aren't born, they're made,
Rich
Intern
Intern
Joined: 24 Nov 2016
Posts: 1
Own Kudos [?]: [0]
Given Kudos: 0
Send PM
Re: The next number in a certain sequence is defined by multiply [#permalink]
But what if a1=-1 and K=2? or a1=0?
GMAT Club Legend
GMAT Club Legend
Joined: 19 Dec 2014
Status:GMAT Assassin/Co-Founder
Affiliations: EMPOWERgmat
Posts: 21846
Own Kudos [?]: 11669 [0]
Given Kudos: 450
Location: United States (CA)
GMAT 1: 800 Q51 V49
GRE 1: Q170 V170
Send PM
Re: The next number in a certain sequence is defined by multiply [#permalink]
Expert Reply
Hi ldelcerro,

While it's certainly beneficial to think about all the possibilities (including negatives and 0), neither of those options 'fits' the given information in the two Facts. If the 9th term is 81, then the first term cannot be negative and it cannot be 0 (since repeatedly multiplying either of those options by a positive constant will NEVER lead to +81). The same issue occurs if the 5th term is 1. Thus, that initial term MUST be a positive.

GMAT assassins aren't born, they're made,
Rich
Current Student
Joined: 14 Nov 2016
Posts: 1174
Own Kudos [?]: 20719 [0]
Given Kudos: 926
Location: Malaysia
Concentration: General Management, Strategy
GMAT 1: 750 Q51 V40 (Online)
GPA: 3.53
Send PM
The next number in a certain sequence is defined by multiply [#permalink]
Bunuel wrote:
The next number in a certain sequence is defined by multiplying the previous term by some positive constant k, where k ≠ 1. How many of the first nine terms in this sequence are greater than 1?

Given that:
\(a_2=a_1*k\)
\(a_3=a_1*k^2\)
\(a_4=a_1*k^3\)
...
\(a_n=a_1*k^{n-1}\)

Also given that k>0 and n=9.

(1) The ninth term in this sequence is 81 --> \(a_9=a_1*k^8=81\). If \(a_1=1\), then all but \(a_1\) will be greater than 1, but if \(a_1=2\), then all will be greater than 1. Not sufficient.

(2) The fifth term in this sequence is 1 --> \(a_5=a_1*k^4=1\). Now, if \(a_1<1\), then \(k>1\), and all terms from \(a_5\) (\(a_6\), \(a_7\), \(a_8\), and \(a_9\)), so 4 terms will be greater than 1 AND if \(a_1>1\), then \(k<1\), and all terms till \(a_5\) (\(a_2\), \(a_3\), \(a_4\), and \(a_5\)), so again 4 terms will be greater than 1. Sufficient.

Answer: B.


Hi Bunuel,

Could you help to elaborate the highlighted part by giving an example to illustrate?
Math Expert
Joined: 02 Sep 2009
Posts: 92948
Own Kudos [?]: 619276 [0]
Given Kudos: 81609
Send PM
Re: The next number in a certain sequence is defined by multiply [#permalink]
Expert Reply
hazelnut wrote:
Bunuel wrote:
The next number in a certain sequence is defined by multiplying the previous term by some positive constant k, where k ≠ 1. How many of the first nine terms in this sequence are greater than 1?

Given that:
\(a_2=a_1*k\)
\(a_3=a_1*k^2\)
\(a_4=a_1*k^3\)
...
\(a_n=a_1*k^{n-1}\)

Also given that k>0 and n=9.

(1) The ninth term in this sequence is 81 --> \(a_9=a_1*k^8=81\). If \(a_1=1\), then all but \(a_1\) will be greater than 1, but if \(a_1=2\), then all will be greater than 1. Not sufficient.

(2) The fifth term in this sequence is 1 --> \(a_5=a_1*k^4=1\). Now, if \(a_1<1\), then \(k>1\), and all terms from \(a_5\) (\(a_6\), \(a_7\), \(a_8\), and \(a_9\)), so 4 terms will be greater than 1 AND if \(a_1>1\), then \(k<1\), and all terms till \(a_5\) (\(a_2\), \(a_3\), \(a_4\), and \(a_5\)), so again 4 terms will be greater than 1. Sufficient.

Answer: B.


Hi Bunuel,

Could you help to elaborate the highlighted part by giving an example to illustrate?


If \(a_1=1\), then from \(a_9=a_1*k^8=81\) --> \(1*k^8=81\) --> \(k=\sqrt{3}\) --> \(a_2=a_1*k=\sqrt{3}>1\) and so on.

If \(a_1=2\), then from \(a_9=a_1*k^8=81\) --> \(2*k^8=81\) --> \(k \approx 1.6\) --> \(a_2=a_1*k \approx 3.3 > 1\) and so on.
Current Student
Joined: 14 Nov 2016
Posts: 1174
Own Kudos [?]: 20719 [0]
Given Kudos: 926
Location: Malaysia
Concentration: General Management, Strategy
GMAT 1: 750 Q51 V40 (Online)
GPA: 3.53
Send PM
The next number in a certain sequence is defined by multiply [#permalink]
Bunuel wrote:
The next number in a certain sequence is defined by multiplying the previous term by some positive constant k, where k ≠ 1. How many of the first nine terms in this sequence are greater than 1?

Given that:
\(a_2=a_1*k\)
\(a_3=a_1*k^2\)
\(a_4=a_1*k^3\)
...
\(a_n=a_1*k^{n-1}\)

Also given that k>0 and n=9.

(1) The ninth term in this sequence is 81 --> \(a_9=a_1*k^8=81\). If \(a_1=1\), then all but \(a_1\) will be greater than 1, but if \(a_1=2\), then all will be greater than 1. Not sufficient.

(2) The fifth term in this sequence is 1 --> \(a_5=a_1*k^4=1\). Now, if \(a_1<1\), then \(k>1\), and all terms from \(a_5\) (\(a_6\), \(a_7\), \(a_8\), and \(a_9\)), so 4 terms will be greater than 1 AND if \(a_1>1\), then \(k<1\), and all terms till \(a_5\) (\(a_2\), \(a_3\), \(a_4\), and \(a_5\)), so again 4 terms will be greater than 1. Sufficient.

Answer: B.


Hi Bunuel, I think \(a_5\) should not count in the term as in highlighted part since \(a_5=1\). If \(a_1>1\), \(a_5=1\), then (\(a_1\), \(a_2\), \(a_3\), \(a_4\)) will be > 1.

Quote:
If \(a_1=2\), then from \(a_9=a_1*k^8=81\) --> \(2*k^8=81\) --> \(k \approx 1.6\) --> \(a_2=a_1*k \approx 3.3 > 1\) and so on.


How could we calculate the value of k without using the calculator? \(2*k^8=81\) --> \(k^8=40.5\) --> \(k \approx 1.6\)
Math Expert
Joined: 02 Sep 2009
Posts: 92948
Own Kudos [?]: 619276 [0]
Given Kudos: 81609
Send PM
The next number in a certain sequence is defined by multiply [#permalink]
Expert Reply
hazelnut wrote:
Quote:
If \(a_1=2\), then from \(a_9=a_1*k^8=81\) --> \(2*k^8=81\) --> \(k \approx 1.6\) --> \(a_2=a_1*k \approx 3.3 > 1\) and so on.


How could we calculate the value of k without using the calculator? \(2*k^8=81\) --> \(k^8=40.5\) --> \(k \approx 1.6\)


We don't need exact value. From \(2*k^8=81\) it should be clear that k will be greater than 1, which is basically what we want to know. All further conclusions could be made based only on this.
Intern
Intern
Joined: 18 Apr 2013
Posts: 27
Own Kudos [?]: 160 [0]
Given Kudos: 3
Send PM
The next number in a certain sequence is defined by multiply [#permalink]
Bunuel wrote:
The next number in a certain sequence is defined by multiplying the previous term by some positive constant k, where k ≠ 1. How many of the first nine terms in this sequence are greater than 1?

Given that:
\(a_2=a_1*k\)
\(a_3=a_1*k^2\)
\(a_4=a_1*k^3\)
...
\(a_n=a_1*k^{n-1}\)

Also given that k>0 and n=9.

(1) The ninth term in this sequence is 81 --> \(a_9=a_1*k^8=81\). If \(a_1=1\), then all but \(a_1\) will be greater than 1, but if \(a_1=2\), then all will be greater than 1. Not sufficient.

(2) The fifth term in this sequence is 1 --> \(a_5=a_1*k^4=1\). Now, if \(a_1<1\), then \(k>1\), and all terms from \(a_5\) (\(a_6\), \(a_7\), \(a_8\), and \(a_9\)), so 4 terms will be greater than 1 AND if \(a_1>1\), then \(k<1\), and all terms till \(a_5\) (\(a_2\), \(a_3\) and \(a_4\)), so again 4 terms will be greater than 1. Sufficient.

Answer: B.



Hi Bunuel, I don't quite understand the explanation for statement 2. You mention " if a1>1, then k<1, and all terms till a5(a2, a3 and a4), so again 4 terms will be greater than 1. Sufficient."
Why would K be <1? In the question it is clearly stated that K is a positive constant and does not equal 1. Thus, K will always be more than 1.

Please correct me if I am wrong. Thanks!
Math Expert
Joined: 02 Sep 2009
Posts: 92948
Own Kudos [?]: 619276 [0]
Given Kudos: 81609
Send PM
Re: The next number in a certain sequence is defined by multiply [#permalink]
Expert Reply
roastedchips wrote:
Bunuel wrote:
The next number in a certain sequence is defined by multiplying the previous term by some positive constant k, where k ≠ 1. How many of the first nine terms in this sequence are greater than 1?

Given that:
\(a_2=a_1*k\)
\(a_3=a_1*k^2\)
\(a_4=a_1*k^3\)
...
\(a_n=a_1*k^{n-1}\)

Also given that k>0 and n=9.

(1) The ninth term in this sequence is 81 --> \(a_9=a_1*k^8=81\). If \(a_1=1\), then all but \(a_1\) will be greater than 1, but if \(a_1=2\), then all will be greater than 1. Not sufficient.

(2) The fifth term in this sequence is 1 --> \(a_5=a_1*k^4=1\). Now, if \(a_1<1\), then \(k>1\), and all terms from \(a_5\) (\(a_6\), \(a_7\), \(a_8\), and \(a_9\)), so 4 terms will be greater than 1 AND if \(a_1>1\), then \(k<1\), and all terms till \(a_5\) (\(a_2\), \(a_3\) and \(a_4\)), so again 4 terms will be greater than 1. Sufficient.

Answer: B.



Hi Bunuel, I don't quite understand the explanation for statement 2. You mention " if a1>1, then k<1, and all terms till a5(a2, a3 and a4), so again 4 terms will be greater than 1. Sufficient."
Why would K be <1? In the question it is clearly stated that K is a positive constant and does not equal 1. Thus, K will always be more than 1.

Please correct me if I am wrong. Thanks!


We are told that k is positive constant and k ≠ 1. So, 0 < k < 1 or k > 1. Nowhere we are told that k is an integer.
Manhattan Prep Instructor
Joined: 04 Dec 2015
Posts: 935
Own Kudos [?]: 1541 [5]
Given Kudos: 115
GMAT 1: 790 Q51 V49
GRE 1: Q170 V170
Send PM
Re: The next number in a certain sequence is defined by multiply [#permalink]
4
Kudos
1
Bookmarks
Expert Reply
akijuneja wrote:
The next number in a certain sequence is defined by multiplying the previous term by some positive constant k, where k ≠ 1. How many of the first nine terms in this sequence are greater than 1?

(1) The ninth term in this sequence is 81.

(2) The fifth term in this sequence is 1


I'm going to try to explain this one in a less 'mathematical' way, as an alternative for those of us who aren't super enthusiastic about algebra. Sequences are also pretty easy to translate into plain English, and that's how I prefer to think about them.

The question stem says that we're going to build a sequence by starting at some number, then multiplying by the same value over and over until we have 9 terms. For example, this would fit the rules:

1, 3, 9, 27, 81, 243, ... etc.

This would also fit the rules:

4, 2, 1, 1/2, 1/4, 1/8, 1/16, 1/32, ... etc.

This would not fit the rules, since k can't equal 1 (we can't just multiply by 1 over and over):

3, 3, 3, 3, 3, 3, ...

This also wouldn't work, since we have to multiply by the same value instead of adding the same value:

1, 2, 3, 4, 5, ...

So that's what the question is telling us. Notice that it's not telling us what the first number in the sequence is, so that could be literally anything. The first number could even be negative! (We just can't multiply by a negative number to get the next term.)

Finally, the question itself asks how many of the first 9 terms are greater than 1. I'm not sure how to figure that out mathematically, to be honest, so I'm going to focus on case testing.

Now, the statements.

Statement 1 The ninth term is 81.

_ _ _ _ _ _ _ _ 81

I know that I started with some value (which I don't know) and then multiplied by the same number over and over, until I hit 81 after multiplying 8 times.

In order to go backwards, I'll have to start with the 81, and divide by the same number, over and over.

Suppose that k = 3. Build the sequence backwards:

_ _ _ _ _ _ _ 27 81

_ _ _ _ _ _ 9 27 81

_ _ _ _ _ 3 9 27 81

_ _ _ _ 1 3 9 27 81

The other four terms will be less than 1. So the answer to the question is "4 terms".

Now let's try a really crazy case. Suppose that k = 81. That is, we'll be dividing by 81 over and over.

_ _ _ _ _ _ _ 1 81

_ _ _ _ _ _ 1/81 1 81

1/81 is less than 1, and so are the other 6 terms at the beginning. The answer the question is "7 terms".

This statement is not sufficient.

Statement 2: The fifth term of the sequence is 1. So, it looks like this:

_ _ _ _ 1 _ _ _ _

We'll be going in both directions now. To go to the left, we'll divide by k. To go to the right, we'll multiply by k.

Let's say that k = 2.

_ _ 1/4 1/2 1 2 4 _ _

Interesting. The four to the left are less than 1. The answer to the question is "4".

Let's say that k = 1/2.

_ _ 4 2 1 1/2 1/4 _ _

Okay, now the four to the right are less than 1. The answer to the question is "4".

Will that always be true? Yeah - if k is a fraction, then the four numbers to the right will all be smaller than 1. If k isn't a fraction, the four numbers to the left will all be smaller than 1. That's because we're either dividing 1 by a number repeatedly (making it smaller), or multiplying 1 by a number repeatedly (making it bigger).

That's why this statement is sufficient, even if k is a fraction.
Target Test Prep Representative
Joined: 14 Oct 2015
Status:Founder & CEO
Affiliations: Target Test Prep
Posts: 18767
Own Kudos [?]: 22065 [1]
Given Kudos: 283
Location: United States (CA)
Send PM
Re: The next number in a certain sequence is defined by multiply [#permalink]
1
Kudos
Expert Reply
akijuneja wrote:
The next number in a certain sequence is defined by multiplying the previous term by some positive constant k, where k ≠ 1. How many of the first nine terms in this sequence are greater than 1?

(1) The ninth term in this sequence is 81.

(2) The fifth term in this sequence is 1


We are given that the next number in a certain sequence is defined by multiplying the previous term by some positive constant k, where k ≠ 1, and we need to determine how many of the first nine terms in this sequence are greater than 1.

Statement One Alone:

The ninth term in this sequence is 81.

Without knowing any other terms in the sequence or the value of the positive constant k we are multiplying, we can’t determine the number of terms in the sequence that are greater than 1. For example, if k = 3, then by going backward we have:

9th term = 81, 8th term = 27, 7th term = 9, 6th term = 3, 5th term = 1, 4th term = 1/3, and so on.

In this case, we have 4 terms that are greater than 1.

However, if k = 9, then by going backward again we have:

9th term = 81, 8th term = 9, 7th term = 1, 6th term = 1/9, and so on.

In this case we have only 2 terms that are greater than 1. Therefore, statement one alone is not sufficient. We can eliminate answer choices A and D.

Statement Two Alone:

The fifth term in this sequence is 1.

Since there are 9 total terms, we see that the 5th term is the middle term in our sequence. In other words there are 4 terms below the 5th term and 4 terms above the 5th term. Since we know that k is positive, it’s either a positive proper fraction or a number greater than 1. Thus, regardless of whether k is a positive proper fraction or a number greater than 1, there will be 4 numbers above 1 and 4 numbers below 1.

For instance, if k = 1/2, then the 1st to the 4th terms, inclusive, are greater than 1, and if k = 2, then the 6th to the 9th terms, inclusive, are greater than 1. Either way, there are 4 terms greater than 1. Statement two alone is sufficient to answer the question.

Answer: B
Manager
Manager
Joined: 19 Aug 2016
Posts: 56
Own Kudos [?]: 6 [0]
Given Kudos: 30
Send PM
Re: The next number in a certain sequence is defined by multiply [#permalink]
Bunuel wrote:
The next number in a certain sequence is defined by multiplying the previous term by some positive constant k, where k ≠ 1. How many of the first nine terms in this sequence are greater than 1?

Given that:
\(a_2=a_1*k\)
\(a_3=a_1*k^2\)
\(a_4=a_1*k^3\)
...
\(a_n=a_1*k^{n-1}\)

Also given that k>0 and n=9.

(1) The ninth term in this sequence is 81 --> \(a_9=a_1*k^8=81\). If \(a_1=1\), then all but \(a_1\) will be greater than 1, but if \(a_1=2\), then all will be greater than 1. Not sufficient.

(2) The fifth term in this sequence is 1 --> \(a_5=a_1*k^4=1\). Now, if \(a_1<1\), then \(k>1\), and all terms from \(a_5\) (\(a_6\), \(a_7\), \(a_8\), and \(a_9\)), so 4 terms will be greater than 1 AND if \(a_1>1\), then \(k<1\), and all terms till \(a_5\) (\(a_2\), \(a_3\) and \(a_4\)), so again 4 terms will be greater than 1. Sufficient.

Answer: B.


Hi Bunuel,

Why are we even considering k<1 as the question explicitly says pos constant...?

Also can u please give me an example where k>1 and a>1 gives a^5=1
Math Expert
Joined: 02 Sep 2009
Posts: 92948
Own Kudos [?]: 619276 [0]
Given Kudos: 81609
Send PM
Re: The next number in a certain sequence is defined by multiply [#permalink]
Expert Reply
zanaik89 wrote:
Bunuel wrote:
The next number in a certain sequence is defined by multiplying the previous term by some positive constant k, where k ≠ 1. How many of the first nine terms in this sequence are greater than 1?

Given that:
\(a_2=a_1*k\)
\(a_3=a_1*k^2\)
\(a_4=a_1*k^3\)
...
\(a_n=a_1*k^{n-1}\)

Also given that k>0 and n=9.

(1) The ninth term in this sequence is 81 --> \(a_9=a_1*k^8=81\). If \(a_1=1\), then all but \(a_1\) will be greater than 1, but if \(a_1=2\), then all will be greater than 1. Not sufficient.

(2) The fifth term in this sequence is 1 --> \(a_5=a_1*k^4=1\). Now, if \(a_1<1\), then \(k>1\), and all terms from \(a_5\) (\(a_6\), \(a_7\), \(a_8\), and \(a_9\)), so 4 terms will be greater than 1 AND if \(a_1>1\), then \(k<1\), and all terms till \(a_5\) (\(a_2\), \(a_3\) and \(a_4\)), so again 4 terms will be greater than 1. Sufficient.

Answer: B.


Hi Bunuel,

Why are we even considering k<1 as the question explicitly says pos constant...?

Also can u please give me an example where k>1 and a>1 gives a^5=1


1. We are told that k is positive constant and k ≠ 1. Nowhere we are told that k is an integer. So, 0 < k < 1 or k > 1.

2. I guess, you are talking about the second statement. \(a_5=a_1*k^4=1\) cannot be true if both k and a1 are greater than 1. That's why the solution considers two cases: \(a_1<1\), \(k>1\) and \(a_1>1\), \(k<1\).
User avatar
Non-Human User
Joined: 09 Sep 2013
Posts: 32697
Own Kudos [?]: 822 [0]
Given Kudos: 0
Send PM
Re: The next number in a certain sequence is defined by multiply [#permalink]
Hello from the GMAT Club BumpBot!

Thanks to another GMAT Club member, I have just discovered this valuable topic, yet it had no discussion for over a year. I am now bumping it up - doing my job. I think you may find it valuable (esp those replies with Kudos).

Want to see all other topics I dig out? Follow me (click follow button on profile). You will receive a summary of all topics I bump in your profile area as well as via email.
GMAT Club Bot
Re: The next number in a certain sequence is defined by multiply [#permalink]
Moderator:
Math Expert
92948 posts

Powered by phpBB © phpBB Group | Emoji artwork provided by EmojiOne